9
$\begingroup$

Let $S\subset\mathbb{R}$ be a convex set and $\mathbb{S}^{n}$ be the set of real symmetric matrices of order $n\times n$.

A matrix valued function $\Gamma: S \rightarrow \mathbb{S}^{n}$ is said to be convex if for all $x_1,x_2 \in S$ and for all $\lambda \in (0,1)$ one has $$\Gamma\left(\lambda x_{1}+\left(1-\lambda\right)x_{2}\right)\preceq\lambda\Gamma\left(x_{1}\right)+\left(1-\lambda\right)\Gamma\left(x_{2}\right),$$ where $\prec$ denotes the Loewner partial order, i.e. $A \prec B$ if $A - B$ is negative definite.

Is the matrix valued function $f: [0,c] \rightarrow \mathbb{S}^{n}$ given by $$ f(t) = e^{At} e^{A^{T} t} $$ convex for any $A \in \mathbb{R}^{n\times n}$ and $c>0$?

Numerical experiments suggest that this statement is true, but I could not prove.

$\endgroup$
6
  • 2
    $\begingroup$ Although the original claim is false (Mike Jury has an excellent explicit counterexample), a much weaker related claim is true. Claim. Let $A \in M_n(\mathbb{C})$. The function $f: [0,c] \to \mathbb{R}_+$ defined by \begin{equation*} f(t) = \|e^{tA}\| \end{equation*} is convex for any unitarily invariant norm $\|\cdot\|$. $\endgroup$
    – Suvrit
    May 13, 2015 at 18:01
  • $\begingroup$ Thanks Suvrit, this claim is also interesting to know :) $\endgroup$
    – Tadashi
    May 13, 2015 at 18:58
  • 2
    $\begingroup$ For completeness I suppose it's worth noting that the claim is true if $A$ is normal. $\endgroup$
    – Noah Stein
    May 13, 2015 at 20:11
  • $\begingroup$ @NoahStein Thanks for the comment. Now I'm wondering if $A$ diagonalizable could imply the claim... $\endgroup$
    – Tadashi
    May 13, 2015 at 21:54
  • 1
    $\begingroup$ Actually, on further thought there is a simple necessary and sufficient condition for $f$ to be convex, I will edit my answer to describe it. $\endgroup$
    – Mike Jury
    May 13, 2015 at 23:58

1 Answer 1

12
$\begingroup$

I think the $3\times 3$ Jordan block $$ A=\begin{pmatrix} 0 & 1 & 0 \\ 0 & 0 & 1 \\ 0 & 0 & 0\end{pmatrix} $$ is a counterexample.

For this $A$, we have $$ \exp{At}=\begin{pmatrix} 1 & t & \frac{t^2}{2} \\ 0 & 1 & t \\ 0 & 0 & 1\end{pmatrix} $$ so $$ f(t) = \begin{pmatrix} 1+t^2+\frac{t^4}{4} & t+\frac{t^3}{2} & \frac{t^2}{2} \\ t+\frac{t^3}{2} & 1+t^2 & t \\ \frac{t^2}{2} & t & 1\end{pmatrix} $$ Take any $c>0$. The last column of $\frac{1}{2}(f(c)+f(0)) - f(\frac{c}{2})$ is $$ \begin{pmatrix} \frac{c^2}{8} \\ 0 \\ 0\end{pmatrix} $$ so the difference $\frac{1}{2}(f(c)+f(0)) - f(\frac{c}{2})$ cannot be positive for any $c$.

EDIT: In fact it is possible to obtain a necessary and sufficient condition for the convexity of $f$ in terms of $A$. Namely, I claim that if the matrix $$ A^2 +A^{\intercal 2}+ 2A A^\intercal $$ is positive semidefinite, then $f$ is convex on $\mathbb R$, and if not, then $f$ is not convex on any interval $[a,b]$. To see this, one can first check that $f(t)$ is convex on $[a,b]$ if and only if the scalar functions $$ f_u(t):=\langle f(t)u, u\rangle $$ are convex on this interval for all $u\in\mathbb R^n$. But since the functions $f_u$ are smooth, they will be convex if and only if the second derivatives are nonnegative. Computing we find $$ f_u^{\prime\prime}(t) = \langle (A^2 +A^{\intercal 2} + 2AA^\intercal)\exp(tA^\intercal)u, \exp(tA^\intercal)u\rangle $$ and the claim follows.

$\endgroup$
1
  • $\begingroup$ Excellent! Thanks for the clear answer! :) $\endgroup$
    – Tadashi
    May 13, 2015 at 18:59

Your Answer

By clicking “Post Your Answer”, you agree to our terms of service and acknowledge you have read our privacy policy.

Not the answer you're looking for? Browse other questions tagged or ask your own question.